Download as pdf or txt
Download as pdf or txt
You are on page 1of 66

Chapter 14

1. Which of the following is true concerning standard costs?


a. Standard costs are estimates of costs attainable only under the most ideal conditions
but rarely practicable.
b. Standard costs are difficult to use with the process costing system.
c. If probably used, standard can help motivate employees.
d. Unfavorable variances, material in amount, should be investigated but large
favorable variances need not be investigated
2. Which of the following terms is best identified with a system of standard costs?
a. Marginal costing
b. Contribution approach
c. Management by exception
d. Standardized accounting system.
3. Which of the following is standard cost nearly like?
a. Estimate cost
b. Budgeted cost
c. Product cost
d. Period cost
4. Which of the following best characterizes a standard cost system?
a. Standard costs involve cost control which, in return, means cost reduction.
b. Standards can pinpoint responsibility and can help motivate employees.
c. All variances in standard should be reviewed
d. All significant unfavorable variances should be reviewed, but significant favorable
variances need not be reviewed.
5. Standard costing will provide the same results as actual or conventional costing when
standard cost variances are distributed to:
a. Cost of goods sold and inventory
b. A statement of financial position account.
c. An income and expense account.
d. None of the above.
6. When performing input-output analysis in standard costing, “standard hours allowed” is a
means of measuring
a. Standard output at standard hour
b. Actual output at standard hour
c. Standard output at actual hour
d. Actual output at actual hour
7. The operating capacity that is required to satisfy anticipated sales demand is
a. Normal capacity
b. Ideal capacity
c. Practical capacity
d. Theoretical capacity
8. If the actual amount of direct material used in aprocess exceeded the standard amounttt
of materials, there was:
a. An unfavorable material price variances
b. A favorable material price variances
c. An unfavorable material quantity variances
d. A favorable material quantity variances
9. When computing variances from standard costs, the difference between the actual and
standard prices multiplied by actual quantity yields a:
a. Combined price-quantity variance.
Chapter 14

b. Price variance
c. Quantity variance
d. Mix variance
10. In standard costs system, the material price variance is obtained by multiplying the
a. Actual price by the difference between the actual quantity purchased and standard
quantity used.
b. Actual quantity purchased by the difference between the actual price and standard
price.
c. Standard price by the difference between standard quantity purchased and the
standard quantity used
d. Standard quantity purchased by the difference between actual price and standard
price.
11. Supposed a standard cost system in being used. What do you call the variation in the
use of material which can be calculated by comparing the record of material withdrawn
with the standard consumption?
a. Volume variance
b. Quantity variance
c. Efficiency variance
d. Price variance
12. What type of material variances for price and usage will arise if the actual number of
pound of materials used exceeds standard pounds allowed but the actual cost was less
than standard cost?
Usage price
a. Unfavorable favorable
b. Favorable favorable
c. Favorable unfavorable
d. Unfavorable unfavorable
13. How should a usage variance that is significant in amount be treated at the end of the
accounting period?
a. Report as deferred charge or credit
b. Allocated among work in process inventory, finished goods inventory and cost of
goods sold.
c. When material is purchased
d. When purchased order is originated
14. If a company follows a practice of isolating variances at the earliest point in time, what
would be the appropriate time to isolate and recognize a direct material price variance?
a. When material is issued
b. When material is used in production
c. When material is purchased
d. When purchase order is originated
15. An unfavorable price variance occur because of
a. Price increases on raw materials
b. Price decreases on raw material
c. Less than anticipated levels of waste in the manufacturing process.
d. More than anticipated level of waste in the manufacturing process.
16. Which department is customarily held responsible for an unfavorable material usage
variance?
a. Quality control
b. Purchasing
Chapter 14

c. Engineering
d. Production
17. The standard unit cost is used in the calculation of which of the following variances?

Material price variance material usage variance


a. No No
b. No Yes
c. Yes No
d. Yes Yes
18. The formula for labor rate variance is
a. Actual hour work x actual hourly rate less standard hourly rate
b. Actual hour work x standard hourly rate less actual hourly rate
c. Standard hourly rate xx standard labor hour less actual hours worked
d. Standard hourly rate x difference in hours
19. Excess direct labor wages variance resulting from overtime premium will be disclosed in
which type of variances?
a. Yield
b. Quantity
c. Labor efficiency
d. Labor rate
20. How is labor rate variance computed?
a. The difference between the standard and actual rate multiplied by standard hour
b. The difference between the standard and actual rate multiplied by actual hour
c. The difference between the standard and actual hour multiplied by actual hour
d. The difference between the standard and actual hour multiplied by the difference
between standard and actual rate.
21. Which of the following statement concerning standard cost is false?- starting here sa
roque
a. If properly used, standards can help to motivate employees
b. All variances, whether favorable or unfavorable, should be investigated
c. Standard costs should be attainable under the condition of efficient operation
d. A standard cost system may be used with a process costing system or job order
system
22. Variance analysis should be used
a. As the only source of information for the performance evaluation
b. To understand why variances arise.
c. To encourage employees to focus on meeting standards.
d. To administer appropriate disciplinary action to employees that do not meet
standards
23. Standard costing system may be used with
a. Just-in-time systems
b. Computer-integrated manufacturing system
c. Total quality management
d. All of the above
24. Standard costing is used to isolate the variances between standard costs and actual
costs. It allows management to measure performance and correct inefficiencies, thereby
helping to
a. Allocate cost accurately
b. Determine the break-even point
Chapter 14

c. Control cost
d. Eliminate management need for subjective decisions
25. Both standard cost and budgeted cost are used for controlling cost. However, the two
terms are not the same. Standard costs differ from the budgeted costs in the standard
costs
a. Are based on engineering studies while budgeted costs are historical cost
b. Costs that were incurred for actual production, while budgeted costs are costs that
should have been incurred for such production
c. Are costs that should have been incurred for actual production, while budgeted costs
are costs that should be incurred for budgeted or planned production?
d. Are always expressed in total amounts, while budgeted costs are always expressed
in per unit amounts
26. Which of the following statement is correct?
a. A standard cost system can never be used in both the job order system and process
costing system
b. Standard cost system can be used in job order costing system, but not in process
costing system
c. Standard costing can be used in either the job order costing and process costing
system
d. Standard cost system can be used in process costing system, but not in job order
costing system
27. In a process costing system, equivalent units of production are computed to determine
the number of complete units that would have been produced, given no beginning and
ending work in process inventories. If a company uses standard costing in the process
costing system, the equivalent unit of production
a. Are multiplied by the standard cost per unit to compute the total standard cost of
units produced.
b. Are never used
c. Are converted to standard equivalent units and the multiplied by the actual cost per
unit
d. Are assumed to be zero.
28. A variance shows a deviation of actual result from standard or budgeted result. In
deciding whether to investigate a variance or not, management may consider the
following except
a. The amount of variance and cost of investigation
b. Whether the variance is favorable or unfavorable
c. The possibility that investigation will eliminate future occurrence of the variance
d. The trend of the variances over time.
29. The following describes the ideal standards, except
a. currently attainable standard
b. theoretical or maximum-efficiency standards
c. make no allowance for waste, machine downtime, and spoilage
d. perfection standards
30. Which of the following does not describe practical standards?
a. Currently attainable standards
b. Can be used for product costing and cash budgeting
c. Performance that is reasonably expected to be achieved with an allowance for
normal spoilage, waste and downtime
Chapter 14

d. Negate the need to adjust standards if working conditions change


31. A standard cost is an estimate of what a cost should be under normal operating
conditions. In establishing standard costs, the following organization personnel may be
involved, except
a. Top management
b. Budgetary accountants
c. Quality control personnel
d. Industrial engineer
32. Because of the impact of costs in most of the businesses, standard costing system is
usually not effective unless the company also has a flexible budgeting system. In flexible
budgeting,
a. Standard costs are not used to prepare budgets for multiple activity level
b. Standard costs are never used
c. Variable costs and fixed cost show the same behavior as budget for different activity
levels are prepared
d. A budget for the expected activity level is prepared showing variable and fixed costs
separately
33. In standard costing system, actual cost are compared with standard cost. The difference
or variance is determined, and the responsibility for such variance is assigned or
identified to a particular person or department, in order to
a. Determine who is at fault and render the appropriate punishment
b. Be able to set the correct selling price of the product
c. Use the knowledge about the variances to promote learning and continuous
improvement in the manufacturing operations
d. Trace the variances to the proper inventory account so that they may be valued at
actual costs

34. This management practice involves giving significant attention only to those areas in
which material variances from expectation occurs, that is, giving less attention on areas
operating as expected.
a. Responsibility accounting
b. Management by objectives
c. Management by exception
d. Material control
35. The material efficiency variances are the difference between actual ang standard
quantities used in production, multiplied by the standard prices. This variance may be
the responsibility of
a. Purchasing department
b. Sales department
c. Production department
d. Personnel department
36. An unfavorable material costing spending variance coupled with a favorable material
efficiency variance would most likely result from
a. The purchase and use of lower than standard quality material
b. The purchase and use of higher than standard quality material
c. Problem involving machine efficiency
d. Changes in product mix
Chapter 14

37. For a recent month, the accountant standard cost variance analysis report showed a
significant amount of unfavorable material efficiency (quantity or usage) variance that
warrants an investigation of this variances should begin with the
a. Personnel management
b. Purchasing manager only
c. Production manager only
d. Production manager or purchasing manager
38. The difference between the actual time used and the amount of the time that should
have been used for actual production, multiplied by the standard labor rate per time is
called
a. Efficiency variance
b. Price variance
c. Spending variance
d. Rate variance
39. The difference between the actual price or rate paid and the standard price or rate that
should have been paid, multiplied by the actual quantity or actual time is called
a. Efficiency variance
b. Quantity variance
c. Time variance
d. Spending variance
40. Which of the following statement is correct?
a. The production manager should be held responsible for the unfavorable rate
variance
b. The labor time variance may be further analyzed by breaking it down into labor mix
and yield variances
c. The personnel manager should be held responsible for the unfavorable rate variance
d. The mix and yield variances analysis is applicable only to material costs.
41. Variable overhead is applied on the basis of standard direct labor hours. If, for a given
period, the direct labor efficiency variance is favorable, the variable overhead efficiency
variance will be
a. Favorable
b. Unfavorable
c. Zero
d. The same amount as the labor efficiency variance
42. If the factory overhead is applied on the basis of output, the variable factory overhead
efficiency variance will be
a. Equal to the direct labor efficiency variance
b. Unfavorable, if the actual production is less than the budgeted production
c. Favorable, if actual production is greater than budgeted production
d. Zero
43. Under two way variance analysis ( two-variance method) for factory overhead, the
difference between the actual factory overhead cost incurred and the factory overhead
applied to actual production is called
a. Volume variance
b. Controllable variance
c. Efficiency variance
d. Total net overhead variances
Chapter 14

44. Under the two way variances method for analyzing factory overhead, which of the
following variances is composed of both variable ang fixed overhead elements?
a. Controllable or budget variance only
b. Volume or capacity variance only
c. Both controllable and capacity variance
d. Neither controllable and capacity variance
45. Rocela Company uses the two way variances method for analyzing its factory overhead
costs. It allocates factory overhead based on 100% of practical capacity. Almost always,
unfavorable variances are reported on the coming period, Rocela Company’s
management is planning to change the denominator level for allocating factory overhead
from 100% to 80%. If this changes were effected, which of the following variances would
be affected?
Controllable Volume
a. No No
b. Yes Yes
c. No Yes
d. Yes No
46. The standard fixed overhead rate is computed on the basis of selected denominator
level, i.g., a predetermined level. If the standard hours allowed for actual production Is
equal to this predetermined activity level for a given period, the volume predetermined
activity level will be
a. Favorable
b. Unfavorable
c. Zero
d. Impossible to compute
47. Which of the following standard cost variances would be least controllable by a
production supervisor?
a. Material efficiency
b. Labor efficiency
c. Variable overhead efficiency
d. Overhead volume
48. Lazy Corporation uses a full absorption standard costing system in accounting for its
production costs. The factory overhead cost is to be applied based on direct labor hours.
Lazy Corporation choice of a production volume or capacity level as a denominator for
calculating its factory overhead rate will affect
a. The fixed factory overhead budget variance
b. The variable overhead application rate
c. The fixed overhead application rate
d. The total amount of budgeted fixed overhead cost
49. Bulsky Corporation will have an overhead volume variance during a period if
a. Its production exceed sales
b. Its actual production is not equal to the predetermined activity level
c. Actual fixed overhead cost is not equal to the budgeted fixed overhead cost
d. Its actual hour worked is not equal to the standard hours allowed for goods output
50. The amount of fixed factory overhead that RRR Corporation was apply to actual
production is the
a. Actual production times the standard time allowed per unit times the standard fixed
overhead application rate
Chapter 14

b. Actual production times the actual time allowed per unit.


c. Actual production times the actual time allowed per unit times the actual fixed
overhead application rate
d. Actual production times the standard time allowed per unit times the actual fixed
overhead application rate.
51. Cost that are not conveniently identified with a particular orders or unit of products.- cost
acctg.
a. Manufacturing overhead
b. Direct labor
c. Direct material
d. Manufacturing costs
52. What costing system is used to avoid any delay in the costing of jobs as experienced in
actual costing?
a. Process costing
b. Job order costing
c. Historical costing
d. Normal costing
53. Costing system that charge the product with the use of an arbitrary overhead application
rate determined at the end of the period:
a. Normal costing
b. Actual costing
c. Job order costing
d. Process costing
54. What costing system is used to allocate overhead with the use of predetermined
overhead rate determined at the beginning of the year based on budgets.
a. Normal costing
b. Actual costing
c. Job order costing
d. Process costing
55. A credit balance in manufacturing overhead control account is
a. Overapplied overhead
b. Underapplied overhead
c. Idle capacity variance
d. Spending variance
56. A debit balance in manufacturing overhead control account is
a. Overapplied overhead
b. Underapplied overhead
c. Idle capacity variance
d. Spending variance
57. Overapplied manufacturing overhead would result if :
a. Manufacturing cot incurred is less than cost charged to production
b. Manufacturing overhead cost incurred were unreasonably large in relation to units
produced
c. Manufacturing overhead cost incurred were greater than cost charged to production
d. The plant were operating at less than normal capacity
58. EG Company found that the differences in product costs resulting from the application
of predetermined overhead costs rate rather than actual overhead rates were very
significant when actual production was substantially less than planned production. The
most likely explanation is that:
Chapter 14

a. Cost of overhead were substantially less than anticipated


b. Overhead was composed chiefly of variable costs
c. Several products were produced simultaneously
d. Fixed manufacturing overhead was a significant cost
59. In highly automated manufacturing, all of the following may be appropriate bases for
manufacturing application except:
a. Machine hours
b. Direct labor hour
c. Number of setups
d. Movement of materials
60. When the amount of overapplied manufacturing overhead is significant, the entry close
overapplied manufacturing overhead will most likely require:
a. A debit to Cost of goods sold
b. Debit to cost of goods sold, finished goods inventories, and work in process
inventory
c. A credit to cost of goods sold
d. Credit to cost of goods sold, finished goods inventory, and work in process inventory
61. The most common treatment of the underapplied overhead at the end of the year would
be to:
a. Carry it as a deferred charges on the balance sheet
b. Report it as a miscellaneous expense on the income statement
c. Debit it to cost of goods sold
d. Prorate between work in process inventory and finish god inventory
62. An objection to the use of predetermined overhead rate based on direct labor cost is
that:
a. These items are difficult to measure
b. The job is charged with more overhead when highly paid operator works on the job
than when a low paid operator performs the work
c. Overhead is allocated in relation to units produced by workers
d. Overhead rates will be distributed inequitable when there are no wage differentials in
the department
63. All of the following phrases are used as alternative terminology for manufacturing
overhead except:
a. Manufacturing expenses
b. Indirect manufacturing cost
c. Factory expense
d. Other expense
64. This variance measures the amount by which actual overhead is more or less than the
amount that should have been spent for the activity level attained:
a. Overhead variance
b. Volume variance
c. Spending variance
d. Price variance
65. This variance is the difference between the actual manufacturing overhead incurred and
the applied manufacturing overhead:
a. Capacity variance
b. Budget Variance
c. Spending variance
d. Material variance
Chapter 14

66. The difference between the actual manufacturing overhead incurred and the budgeted
overhead based on the budget capacity used is.
a. Controllable variance
b. Volume variance
c. Overhead variance
d. Price variance
67. What does a favorable variance represent?
a. The actual overhead costs were less than the costs applied to production
b. The actual overhead costs were more than the costs applied to production
c. The spending variance is more than the volume variance
d. The spending variance is less than the volume variance
68. What type of direct material variances for price and usage will arise if the actual number
of pounds of materials used exceeds standard pounds allowed but actual cost was less
than standard cost?- dayag
Usage price
a. Unfavorable favorable
b. Favorable favorable
c. Favorable Unfavorable
d. Unfavorable unfavorable
69. An unfavorable price variance occurs because of:
a. Price increases on raw materials
b. Price decreases on raw materials
c. Less than anticipated levels of waste in the manufacturing process
d. More than anticipated levels of waste in the manufacturing process
70. If a company follows a practice of isolating variances at the earliest time, what would be
the appropriate time to isolate and recognize direct material price variances?
a. When material is issued
b. When material is purchased
c. When material is used in production
d. When the purchase order is originated
71. Materials usage variances are normally chargeable to which department?
a. Production
b. Purchasing
c. Finished goods
d. Material usage
72. Which of the following would least likely cause an unfavorable material quantity
variance?
a. Materials that do not meet specification
b. Machineries that has not been maintained properly.
c. Labor that possesses skills equal to those required by the standard
d. Scheduling of substantial overtime
73. RL planned to produce 3000 units of its single product, telegram, during November. The
standard specification for one unit of telegram includes 6 pounds of material at P.3 per
pound. Actual production in November was 3100 units of telegram. The accountant
computed a favorable material purchase price variance at P380 and an unfavorable
materials quantity variance at P120. Based on these variances, one would conclude
that
a. More material were purchased that used
b. More material were used than purchased
Chapter 14

c. The actual cost of material is less than the standard cost


d. The actual usage of materials was less than the standard allowed.
74. The sum of variable overhead spending variance and the variable efficiency variance?
a. Total variable overhead variance
b. Expected actual variance
c. Ideal standard
d. Normal standard
75. The difference between actual hours worked and the hours that should have been
worked at the level of production, times the standard variable overhead rate?
a. Variable overhead spending variance
b. Overhead efficiency variance
c. Labor efficiency variance
d. Material usage variance
76. The difference between the actual variable overhead incurred and a performance budget
for variable overhead is?
a. Expected actual variance
b. Material usage variance
c. Variable spending variance
d. Labor efficiency variance
77. The difference between actual hours worked and the hours that should have been
worked at the level of production, times the standard wage rate?
a. Labor efficiency variance
b. Labor rate variance
c. Expected actual variance
d. Total labor variance
78. The difference between the actual hours worked times the standard wage rate and the
actual payroll?
a. Labor efficiency variance
b. Total labor variance
c. Actual labor variance
d. Labor rate variance
79. A measure of how well materials were utilized in the production process?
a. Material usage variance
b. Material price variance
c. Total material variance
d. None of the above
80. The difference between the actual price and the standard price, times the actual quantity
of material purchases?
a. Material usage variance
b. Material price variance
c. Total material variance
d. None of the above
81. An estimate of what will happen?
a. Ideal standard
b. Normal standard
c. Expected overhead variance
d. Expected material variance
82. What should be achieved with normal workers in a normal setting?
a. Ideal standard
b. Theoretical standard
Chapter 14

c. Normal standard
d. Perfect standard
83. What should be achieved if all conditions are perfect?
a. Ideal standard
b. Theoretical standard
c. Perfect standard
d. All of the above
84. It indicates what the cost of the time or the material should be?
a. Quantity standard
b. Cost standard
c. Both a and b
d. Neither a nor b
85. It indicates how much of the cost should be used in manufacturing a unit of a product or
in providing a unit of services
a. Quantity standard
b. Cost standard
c. Both a and b
d. Neither a nor b
86. It indicates what cost should be incurred under attainable, acceptable performance?
a. Quantity standard
b. Cost standard
c. Both a and b
d. Neither a nor b
87. Used as yardsticks that measure achievement or lack of achievement?
a. Budget
b. Standard
c. Normal
d. All of the above
88. The following are benefits of standard costs except.
a. Aid management planning through budgeting
b. Standard cost system integrate coordination of all elements of the organization
towards achieving the same goal
c. It can save data processing system cost
d. None of the above
89. Which of the following is not the responsibility of line supervisor?
a. Volume variance
b. Price variance
c. Quantity variance
d. None of the above
90. The possible causes of material price variance are as follow except?
a. Fluctuations in market prices of materials.
b. Failure to take cash discounts available
c. Unfavorable purchase contract terms
d. None of the above
91. Which of the following is not the possible cause of price variance?
a. Waste and loss of material in handling and processing
b. Purchasing material of substandard quality or in uneconomical lots
c. Purchasing from distant suppliers
d. None of the above
92. Purchasing department is usually responsible for material price variance?
Chapter 14

Production planning department may be responsible for material price variance?


a. Only Statement 1 is true
b. Only Statement 2 is true
c. Both statement are true
d. Both statement are false
93. The reason why production planning department could be possibly responsible for
unfavorable price variances:
a. Because of request for rush orders due to poor scheduling
b. When they specify a certain brand-name materials or materials of certain grade or
quality other than those initially included in the bill of materials
c. Either of A or B
d. Both a and b
94. The following are the possible causes of material usage variance except:
a. Variation in yields from materials
b. Lack of proper tools or machines
c. Substitution of defective or nonstandard materials
d. None of the above
95. Statement1: Waste and loss of materials in handling and processing is one of the causes
of material price variance.
Statement 2: spoilage or production of excess scrap because of inexperienced workers
or poor supervision can cause material price variance.
a. Only statement 1 is true
b. Only statement 2 is true
c. Both statements are true
d. Both statements are false
96. Statement1: Waste and loss of materials in handling and processing is one of the
causes of material usage variance.
Statement 2: spoilage or production of excess scrap because of inexperienced workers
or poor supervision can cause material quantity variance.
a. Only statement 1 is true
b. Only statement 2 is true
c. Both statements are true
d. Both statements are false
97. Statement1: Waste and loss of materials in handling and processing is one of the causes
of material price variance.
Statement 2: spoilage or production of excess scrap because of inexperienced workers
or poor supervision can cause material quantity variance.
a. Only statement 1 is true
b. Only statement 2 s true
c. Both statements are true
d. Both statements are false
98. Statement1: Waste and loss of materials in handling and processing is one of the causes
of material usage variance.
Statement 2: spoilage or production of excess scrap because of inexperienced workers
or poor supervision can cause material price variance.
a. Only statement 1 is true
b. Only statement 2 is true
c. Both statements are true
d. Both statements are false
Chapter 14

99. Statement1: Purchasing from suppliers is one of the causes of material price variance.
Statement2: favorable purchase contract term is one of the causes of material price
variance.
a. Only statement 1 is true
b. Only statement 2 is true
c. Both statements are true
d. Both statements are false
100. Statement1: substitution of defective or nonstandard materials is one of the
causes of price usage variance.
Statement2: favorable purchase contract term is one of the causes of material price
variance.
a. Only statement 1 is true
b. Only statement 2 is true
c. Both statements are true
d. Both statements are false
101. Which one of the following terms best describes the rate of output which qualified
workers can achieve as an average over the working days or shift, without over-exertion,
provided they adhere to the specified method of working and are well motivated in their
work?
a. Standard time
b. Standard hours
c. Standard units
d. Standard performance
102. The best characteristics of a standard cost system is
a. Standard can pinpoint responsibility and help motivate
b. All variances from standard should be reviewed
c. All significant unfavorable variances should be reviewed
d. Standard cost involves cost control which is cost reduction
103. Standard costs are used in the following except:
a. Income determination
b. Controlling costs
c. Measuring efficiencies
d. Forming a basis for price setting
104. Standard cost are least useful for
a. Measuring production efficiencies
b. Simplifying costing procedures
c. Job production systems
d. Determining minimum inventory levels
105. Which of the following is a standard cost nearly like?
a. Estimated cost
b. Budgeted cost
c. Product cost
d. Period cost
106. A difference between standard cost used for cost control and budgeted cost
a. Can exist because standard cost must be determined after the budget is completed
b. Can exist because standard costs represent what cost should be while budgeted
costs represent expected actual cost
c. Can exist because budgeted costs are historical costs while standard costs are
based on engineering studies
Chapter 14

d. Can exist because establishing budgeted costs involves employee participation and
standard costs do not.
107. Normal costing and standard costing differ in that
a. The two systems can show different overhead budgeted variances.
b. Only normal costing can be used with absorption costing
c. The two systems show different volume variances if standard hours do not equal
actual hours
d. Normal costing is less appropriate for multiproduct firms.
108. When standard costs are used in a process-costing system, how, if at all, are
equivalent units of production (EUP) involved or used in the cost report at standard?
a. Equivalent units
b. Equivalent units are computed using a special approach.
c. The actual equivalent units are multiplied by the standard cost per unit
d. The standard equivalent units are multiplied by the actual cost per unit
109. The type of standard that is intended to represent challenging yet attainable
results is:
a. Theoretical standard
b. Flexible budget standard
c. Expected actual standard
d. Controllable cost standard
110. A company using the very tight standards in a standard cost system should
expect that
a. Most variances will be unfavorable
b. No incentive bonus will be paid
c. Cost will be controlled better than if lower standards were used
d. Employees will be strongly motivated to attain the standard
111. A predetermined overhead rate for fixed costs is unlike a standard fixed costs per
unit in that a predetermined overhead rate is
a. Based on an input factor like direct labor hours and a standard cost per unit is based
on a unit of output
b. Based on practical capacity and a standard fixed cost can be based on any level of
activity
c. Used with variable costing while a standard fixed cost is used with absorption costing
d. Likely to be higher than a standard fixed cost per unit
112. If a company wishes to establish factory overhead budget system in which
estimated costs can be derived from estimates of activity levels, it should prepare a
a. Flexible budget
b. Fixed budget
c. Capital budget
d. Discretionary budget
113. Lanta restaurant compares monthly operating results with a static budget. When
actual sales are less than budget. When actual sales are less than budget, would lanta
usually report favorable variances on variable food costs and fixed discretionary
salaries?
Variable food cost fixed supervisory cost
a. Yes Yes
b. Yes No
c. No Yes
d. No No
Chapter 14

114. The primary difference between fixed (static) budget and a variable (flexible)
budget is that a fixed budget:
a. Cannot be changed after the period begin, while a variable budget can be changed
after the period begin
b. Is a plan for a single level of sales ( or other measure of activity); while a variable
budget consists of several plans, one of several levels of sales (or other measure of
activity)
c. Includes only fixed costs; while variable budget includes only variable costs
d. Is concerned only with future acquisitions of fixed assets; while a variable budget is
concerned with expenses that vary with sales
115. Which of the following terms is best identified with a system of standard costs?
a. Contribution approach
b. Management by exception
c. Marginal costing
d. Standard accounting system
116. Which department is typically responsible for material price variances?
a. Engineering
b. Production
c. Purchasing
d. Sales
117. Under a standard cost system, the materials efficiency variances are
responsibility of
a. Production and industrial engineering
b. Purchasing and industrial engineering
c. Purchasing and sales
d. Sales and industrial engineering
118. Which of the following people is most likely responsible for an unfavorable
variable overhead efficiency variance?
a. Production supervisor
b. Accountant agent
c. Supplier
d. Purchasing agent
119. Which of the following is least likely to be affected by hiring workers with less skill
than those already working?
a. Material use variance
b. Labor rate variance
c. Material price variance
d. Variable overhead efficiency variance
120. Which of the following standard costing variances would be least controllable by
a production supervisor?
a. Overhead volume
b. Material usage
c. Labor efficiency
d. Overhead efficiency
121. The variance resulting from obtaining an output different from the one expected
on the basis of input is the:
a. Mix variance
b. Usage variance
c. Yield variance
Chapter 14

d. Efficiency variance
122. For the doughnuts of Mcdonut co. the purchasing manager decided to buy 65000
bags of flour with a quality rating two grades below that which the company normally
purchased. This purchase covered about 90% of the flour equipment for the period. As
to the material variances, what will be the effect?
Price variance usage variance
a. Unfavorable favorable
b. Favorable unfavorable
c. No effect unfavorable
d. Favorable favorable
123. Using the two variance method for analyzing overhead, which of the following
variances contains both variable and fixed overhead elements?
Controllable Volume Efficiency
a. Yes yes yes
b. Yes yes no
c. Yes no no
d. No no no
124. Which of the following unfavorable variances is directly affected by the relative
position if a production process on a learning curve?
a. Material mix
b. Material price
c. Labor rate
d. Labor efficiency
125. A manager prepared the following information by which to analyze labor costs for
the month:
Actual hours @ actual rate Actual hours @ standard rate standard hours @
standard rate
P10000 P9800 P8820
What variance is P980?
a. Labor efficiency variance
b. Labor rate variance
c. Volume variance
d. Labor spending variance
126. A credit balance in the labor efficiency variance indicates that:
a. Standard hours exceeded actual hours
b. actual hours exceeded standard hours
c. standard rate and standard hours exceeded actual rate and actual hours
d. actual rate and actual hours exceeded standard rate and standard hours
127. if the actual labor rate exceeds the standard labor rate and the actual labor hours
exceeds the number of hours allowed, the labor rate variance and Labor efficiency
variance will be
Labor rate variance labor efficiency variance
a. favorable favorable
b. Favorable unfavorable
c. Unfavorable favorable
d. Unfavorable unfavorable
128. In the analysis of standard cost variances, the items which received the most
diverse treatment in accounting is
a. Direct labor cost
Chapter 14

b. Factory overhead cost


c. Direct material cost
d. Variable cost
129. When the expenses estimated for the capacity attained differ from the actual
expenses incurred, the resulting balance is termed the
a. Activity variance
b. Budget variance
c. Unfavorable variance
d. Volume variance
130. The total overhead variance is
a. The difference between the actual overhead cost and budgeted overhead
b. Based on actual hours worked for the units produced
c. The difference between actual overhead costs and applied overhead
d. The difference between budgeted overhead cost and applied overhead
131. Management scrutinizes variances because
a. Management desires to detect such variances to be able to plan for promotions
b. The management needs to determine the benefits foregone by such variances
c. It is desirable under conventional knowledge on good environment
d. Management recognizes the need to know why variances happen to be able to make
corrective actions and fairly reward good performers
132. If a company uses a predetermined rate for absorption of manufacturing
overhead, the volume variance is
a. The under or over applied fixed cost element of overhead
b. The under or over applied variance cost element of overhead
c. The difference between the budgeted cost and actual cost of fixed overhead items
d. The difference between the budgeted cost and actual cost of a variable overhead
items
133. The production volume variance occurs when using the
a. Absorption costing approach because of production exceeding the sales
b. Absorption costing approach because production differs from the used in setting the
fixed overhead rate used in applying fixed overhead to production
c. Variable costing approach because sale exceeding the production for the period
d. Variable costing approach because production exceeding the sales for the period
134. Henley Company uses a standard cost system in which it applies manufacturing
overhead to the units of product on the basis of direct labor hours. For the month of
January, the fixed manufacturing overhead volume variance was 2220 favorable. The
company uses the fixed manufacturing overhead rate of 1.85 per direct labor hour.
During january, the standard direct labor hours allowed for the month’s output:
a. Exceeds denominator hours by 1000
b. Fell short of denominator hours by 1000
c. Exceeded denominator hours by 1200
d. Fell short of denominator hour by 1200
135. A spending variance for variable factory O/h based on direct labor on direct labor
hours is the difference between actual variable factory overhead and the variable factory
overhead that should have been incurred for the actual hours worked. This variance
results from
a. Price and quantity difference for overhead cost
b. Price differences for overhead costs
c. Quantity differences for overhead cost
Chapter 14

d. Differences caused by production volume variation


136. Which of the following is the most probable reason a company would experience
an unfavorable labor rate variance and a favorable efficiency variance?
a. The mix of workers assigned to the particular job was heavily weighted toward the
use of higher-paid, experienced individuals.
b. The mix of workers assigned to the particular job was heavily weighted toward the
use of new relatively low-paid, unskilled workers.
c. Because of the production schedule, workers from other production areas assigned
to assist in this particular process
d. Defective materials caused more labor to be used to product a standard unit.
137. The variable factory overhead rate under the normal volume, practical capacity
and expected activity levels would be the
a. Same except for practical capacity
b. Same except for expected capacity
c. Same except for normal volume
d. Same for all three activity levels.
138. A company reported a significant materials efficiency variance for the month of
January. All of the following are possible explanations for the variance except
a. Cutting back preventive maintenance
b. Inadequately training and supervising the labor force
c. Processing a large number of rush orders
d. Producing more units than planned for in the master budget
139. A debit balance in the labor efficiency variance indicate that
a. Standard hours exceed actual hours.
b. Actual hours exceed standard hours
c. Standard rate exceeds actual rate
d. Actual rate exceeds standard rate
140. What type of direct material variances for price and usage will arise if the actual
number of pounds of materials used was less than standard pounds allowed but actual
cost exceeds standard cost?
Usage price
a. Unfavorable favorable
b. Favorable favorable
c. Favorable unfavorable
d. Unfavorable unfavorable
141. Which of the following would not explain an adverse direct labor efficiency
variance?
a. Poor scheduling of direct labor hours
b. Setting standard efficiency at a level that is too low
c. Unsually lengthy machine breakdowns
d. A reduction I direct labor training
142. You used predetermined overhead rates and the resulting variances when
compared with the results using the actual rates were substantial. Production data
indicated that volume were lower than the plan by a large difference. This situation can
be due to
a. Overhead being substantially composed of fixed costs.
b. Overhead substantially composed of variable costs
c. Overhead costs being recorded as planned
d. Products being simultaneously manufactured in single runs.
Chapter 14

143. During 1990, a department’s three-variance manufacturing O/H standard costing


system reported unfavorable spending variance and volume variances. The activity level
selected for allocating factory overhead to the product was based on 80% of practical
capacity. If 100% of practical capacity had been selected instead, how would the
reported unfavorable spending and volume variances have been affected?
Spending variance volume variance
a. Increased unchanged
b. Increased increased
c. Unchanged increased
d. Unchanged unchanged
144. The journal entry to record the direct materials quantity variance may be
recorded
a. Only when direct materials are purchased
b. Only when direct materials are issued to production
c. Either a or b
d. When inventory is taken at the end of the year.
145. Overapplied factory overhead results when
a. A plant is operated at less than its normal capacity
b. Factory overhead costs incurred are greater than the cost charged to production.
c. Factory overhead costs incurred are less than cost charged to production
d. Factory overhead costs incurred are unreasonably large in relation to the number of
unit produced.
146. Standard costing will result to the same results as actual or conventional costing
when standard cost variance are distributed to
a. Cost of goods sold and inventory
b. A balance sheet account
c. An income or expense account
d. Cost of goods sold
147. A standard cost:
a. Is the true cost of a unit of production
b. Is the budget for the production of one unit of a product or services
c. Can be useful in calculating equivalent units
d. Is normally the average cost within an industry
e. Is almost always the actual cost from previous years
148. Which of the following is the predetermined cost that can be used in the
calculation of a variance?
a. Product cost
b. Actual cost
c. Standard cost
d. Differential cost
e. Marginal cost
149. Variances are computed by taking the difference between which of the following?
a. Product cost and period cost
b. Actual cost and rate factors
c. Price factors and rate factors
d. Actual cost and standard cost
e. Product cost and standard cost
150. What is the best definition of management by exceptions
a. Choosing exceptional managers
Chapter 14

b. Controlling actions of subordinates through acceptance of management techniques


c. Investigating unfavorable variances
d. Devoting management to investigate significant variances
e. Controlling costs so that non-zero variances are treated as exceptional
151. Which of the following are methods in setting standards?
a. Analysis of historical data
b. Task analysis
c. Task analysis and analysis of historical data
d. Matrix application form
e. Goal congruence
152. Which of the following individual in the setting of either direct material standards
or direct labor standards?
a. The purchasing manager
b. A production supervisor
c. An engineer
d. A machine operator
e. A company’s president
153. A perfect standard:
a. Tends to motivate employee over a period of time
b. Is attainable in an ideal operating environment
c. Would make allowance for normal amounts of scrap and waste
d. Is generally preferred by behavioral scientist
e. Will result in a number of favorable variances on a performance report
154. Consider the following statements:
I. Behavioral scientist find that perfection standard often discourage employees
and result in low worker morale
II. Practical standards are also known as attainable standard
III. Practical standards incorporate a certain amount of inefficiency such that caused
by a occasional machine breakdown

Which of the following above statements is (are) true?


a. I only
b. II only
c. III only
d. II and III
e. I,II, and III
155. Which of the following would be considered if a company desires to establish a
series of practical manufacturing standards?
a. Production time lost during unusual machinery breakdowns
b. normal defect rate in an assembly process
c. highly unusual spoilage rates with direct materials
d. Quantity discounts associated with purchases of direct materials
e. Both B and D
156. Which of the following would not be considered if a company desires to establish
a series of practical manufacturing standards?
a. Production time lost during unusual machinery breakdown
b. Normal worker fatigue
c. Freight charges on incoming raw materials
d. Production time lost during setup procedures for new manufacturing runs.
e. The historical 2% defect rates associated with raw material inputs.
Chapter 14

157. Which of the following choices correctly notes a characteristic associated with
perfection standard and one of associated with practical standard?

Perfection standard Practical standard


a. Attainable in an ideal environment Incorporate abnormal occurrences when
setting quantity and efficiency targets
b. Results in many unfavorable variances Are often attainable by workers
c. Tend to boost worker morale Generally preferred by behavioral scientists
d. Generally, are easily achieved by Result in both favorable and unfavorable
workers variances
e. Generally preferred by behavioral Are easier to achieve than perfection
scientists standards
158. Consider the following statements:
I. The standard cost per unit of material is used to calculate a material price
variance
II. The standard cost per unit of materials is used to calculate a material quantity
variance
III. The standard cost per unit of materials cannot be determined until the end of
the period.

Which of the following statement above is (are) true?


a. I only
b. II only
c. III only
d. I and II
e. I, II, and III
159. Which of the following choices correctly notes the use of the standard price per
unit of direct material when calculating the materials price variance and the material
quantity variance?

Price variance Quantity variance


a.
Used Always used
b.
Used Occasionally used
c.
Used Not used
d.
Not used Always used
e.
Not used Not used
160. Most companies base the calculation of the material price variance on the:
a. Number of units purchased
b. Number of units spoiled
c. Number of units that should be used
d. Number of units actually used
e. Number of units to be purchased during the next accounting period.
161. Which of the following correctly lists all the information needed to calculate a
labor rate variance?
a. Standard labor rate and actual hour worked
b. Actual hours worked and actual units produced
c. Standard labor rate, actual labor rate and actual hour worked
d. Actual labor rate, standard labor rate and actual hours worked
e. Actual labor rate, standard labor and actual hours worked
Chapter 14

162. Which of the following variances are most similar with respect to the manner in
which they are calculated?
a. Labor rate variance and labor efficiency variance
b. Materials price variance and materials quantity variance
c. Materials price variance, material quantity variance and total material variance
d. Material price variance and labor efficiency variance
e. Material quantity variance and labor efficiency variance
163. Which of the following variances cannot occur together during the same
accounting period?
a. Unfavorable labor rate variance and favorable labor efficiency variance
b. Unfavorable efficiency variance and favorable material quantity variance
c. Favorable labor rate variance and unfavorable total labor variance
d. Favorable labor efficiency variance and favorable material quantity variance
e. None of the above, as all of these variance combination are possible
164. If a company has an unfavorable direct material quantity variance then,:
a. The direct material price variance is favorable
b. Total direct material variance is unfavorable
c. The total direct material variance is favorable
d. The direct labor efficiency variance is unfavorable
e. Any of the above variance can occur
165. A favorable labor efficiency variance is created when
a. Actual labor hour worked exceed standard hours allowed.
b. Actual hours worked are less than the standard hour allowed
c. Actual wages paid are less than amount that should have been paid
d. Actual unit produced exceed budgeted production levels
e. Actual unit produced exceed standard hours allowed
166. When considering whether to investigate variance, managers shoud consider all
of the following except the variance’s:
a. Size
b. Patterns of recurrence
c. Trend over time
d. Nature, namely, whether it is favorable or unfavorable
e. Controllability
167. Which of the following combination of direct material variances might prompt
management to undertake a detailed variance investigation?
a. Price, unfavorable, quantity, unfavorable
b. Price, unfavorable, quantity , favorable
c. Price, favorable, quantity, unfavorable
d. Price, favorable, quantity, favorable
e. All of the above
168. Considering the following statement about variance investigation:
I. Variance investigation involves a look at only unfavorable variances
II. Variance investigation is typically based on a cost benefit analysis
III. Variance investigation is often performed by establishing guideline similar to the
following investigate variances that are greater than SX or greater than Y% of the
standard cost

Which of the above statements is (are) true?


a. I only
Chapter 14

b. II only
c. III only
d. II and III
e. I, II, and III
169. A statistical control chart is best used for determining:
a. Direct material price variance
b. Direct labor variances
c. Whether a variance is favorable or unfavorable
d. Who should be held accountable for a specific variances
e. Whether a particular variance should be investigated.
170. The individual generally responsible for the direct material variance is the:
a. Sales manager
b. Production manager
c. Purchasing manager
d. Finance manager
e. Head of human resources department
171. A production supervisor generally has little influence over the:
a. Direct material quantity variance
b. Direct labor rate variance
c. Direct labor efficiency variance
d. Direct material price variance
e. Number of units produced
172. In which department would an investigation normally begin regarding an
unfavorable material quantity variance?
a. Quality control
b. Purchasing
c. Engineering
d. Production
e. Receiving
173. Cohen Corporation has a favorable material quantity variance. Which department
would likely be asked to explain the cause of this variance?
a. Engineering
b. Purchasing
c. Production
d. Marketing
e. None, because the variance is favorable
174. Rogers, Inc. had an unfavorable labor efficiency variance and an unfavorable
material quantity variance. Which of the following might be held accountable for these
variances?
a. Purchasing, because bad material can harm labor efficiency
b. Production, because inefficient workers may use more materials tha allowed
c. Purchasing, and/or production
d. Marketing
e. Shipping
175. A direct material quantity variance can be caused by all of the following except:
a. Improper employee training
b. Changes in sales volume
c. Acquisition of materials at a very attractive price
d. Disgruntled workers
176. A direct labor efficiency variances cannot be caused by :
Chapter 14

a. Inexperienced employees
b. Poor quality raw materials
c. Employee inefficiency
d. An out of date labor time standard
e. Producing fewer finished units than originally planned
177. Justin Company recently purchases material from a new supplier at a very
attractive price. the materials were found to be a of poor quality, and the company’s
laborers struggled significantly as they shaped the materials into finished goods. In a
desperation move to make up of some of the time lost, the manufacturing supervisor
brought in more senior employees from another part of the plant. Which of the following
variances would have a high probability of arising from this situation?
a. Material price variance, favorable
b. Material quantity variance, unfavorable
c. Labor rate variance, unfavorable
d. Labor efficiency variance, unfavorable
e. All of the above

178. A standard cost:


A. is the "true" cost of a unit of production.
B. is a budget for the production of one unit of a product or service.
C. can be useful in calculating equivalent units.
D. is normally the average cost within an industry.
E. is almost always the actual cost from previous years.

179. Which of the following is a predetermined estimated cost that can be used in the
calculation of a variance?
A. Product cost.
B. Actual cost.
C. Standard cost.
D. Differential cost.
E. Marginal cost.

180. Variances are computed by taking the difference between which of the following?
A. Product cost and period cost.
B. Actual cost and differential cost.
C. Price factors and rate factors.
D. Actual cost and standard cost.
E. Product cost and standard cost.

181. The term "management by exception" is best defined as:


A. choosing exceptional managers.
B. controlling actions of subordinates through acceptance of management techniques.
C. investigating unfavorable variances.
D. devoting management time to investigate significant variances.
E. controlling costs so that non-zero variances are treated as "exceptional."
Chapter 14

182. Which of the following are methods for setting standards?


A. Analysis of historical data.
B. Task analysis.
C. Task analysis and the analysis of historical data.
D. Matrix application forms.
E. Goal congruence.

183. Which of the following individuals is least likely to become involved in the setting of
either direct material standards or direct labor standards?
A. The purchasing manager.
B. A production supervisor.
C. An engineer.
D. A machine operator.
E. A company's president.

184. A perfection standard:


A. tends to motivate employees over a long period of time.
B. is attainable in an ideal operating environment.
C. would make allowances for normal amounts of scrap and waste.
D. is generally preferred by behavioral scientists.
E. will result in a number of favorable variances on a performance report.

185. Consider the following statements:

I Behavioral scientists find that perfection standards often discourage employees


and result in low worker morale.
II Practical standards are also known as attainable standards.
III Practical standards incorporate a certain amount of inefficiency such as that
caused by an occasional machine breakdown.

Which of the above statements is (are) true?


A. I only.
B. II only.
C. III only.
D. II and III.
E. I, II, and III.

186. Which of the following would be considered if a company desires to establish a series of
practical manufacturing standards?
A. The productivity loss associated with a short-term worker slowdown.
B. Normal defect rates in an assembly process.
C. Highly unusual spoilage rates with direct materials.
D. Quantity discounts associated with purchases of direct materials.
E. Both "B" and "D"

187. Which of the following would not be considered if a company desires to establish a
series of practical manufacturing standards?
A. Production time lost during unusual machinery breakdowns.
Chapter 14

B. Normal worker fatigue.


C. Freight charges on incoming raw materials.
D. Production time lost during setup procedures for new manufacturing runs.
E. The historical 2% defect rate associated with raw material inputs.

188. Which of the following choices correctly notes a characteristic associated with perfection
standards and one associated with practical standards?
Perfection Standards Practical Standards
A. Attainable in an ideal environment Incorporate abnormal occurrences
when setting quantity and efficiency
targets
B. Result in many unfavorable variances Are often attainable by workers
C. Tend to boost worker morale Generally preferred by behavioral
scientists
D. Generally, are easily achieved by Result in both favorable and
workers unfavorable variances
E. Generally preferred by behavioral Are easier to achieve than perfection
scientists standards

189. Consider the following statements:

I The standard cost per unit of materials is used to calculate a materials price
variance.
II The standard cost per unit of materials is used to calculate a materials quantity
variance.
III The standard cost per unit of materials cannot be determined until the end of the
period.

Which of the above statements is (are) true?


A. I only.
B. II only.
C. III only.
D. I and II.
E. I, II, and III.
Chapter 14

190. Which of the following choices correctly notes the use of the standard price per unit of
direct material when calculating the materials price variance and the materials quantity
variance?
Price Variance Quantity Variance
A. Used Always used
B. Used Occasionally used
C. Used Not used
D. Not used Always used
E. Not used Not used

191. Most companies base the calculation of the materials price variance on the:
A. number of units purchased.
B. number of units spoiled.
C. number of units that should have been used.
D. number of units actually used.
E. number of units to be purchased during the next accounting period.

192. Which of the following correctly lists all the information needed to calculate a labor rate
variance?
A. Standard labor rate and actual hours worked.
B. Actual hours worked and actual units produced.
C. Standard labor rate, actual labor rate, and actual units produced.
D. Actual labor rate and actual hours worked.
E. Actual labor rate, standard labor rate, and actual hours worked.

193. Which of the following variances are most similar with respect to the manner in which
they are calculated?
A. Labor rate variance and labor efficiency variance.
B. Materials price variance and materials quantity variance.
C. Materials price variance, materials quantity variance, and total materials variance.
D. Materials price variance and labor efficiency variance.
E. Materials quantity variance and labor efficiency variance.

194. Which of the following variances cannot occur together during the same accounting
period?
A. Unfavorable labor rate variance and favorable labor efficiency variance.
B. Unfavorable labor efficiency variance and favorable materials quantity variance.
C. Favorable labor rate variance and unfavorable total labor variance.
D. Favorable labor efficiency variance and favorable materials quantity variance.
E. None of the above, as all of these variance combinations are possible.
Chapter 14

195. If a company has an unfavorable direct-material quantity variance, then:


A. the direct-material price variance is favorable.
B. the total direct-material variance is unfavorable.
C. the total direct-material variance is favorable.
D. the direct-labor efficiency variance is unfavorable.
E. any of the above variances can occur.

196. A favorable labor efficiency variance is created when:


A. actual labor hours worked exceed standard hours allowed.
B. actual hours worked are less than the standard hours allowed.
C. actual wages paid are less than amounts that should have been paid.
D. actual units produced exceed budgeted production levels.
E. actual units produced exceed standard hours allowed.

197.When considering whether to investigate a variance, managers should consider all of the
following except the variance's:
A. size.
B. pattern of recurrence.
C. trends over time.
D. nature, namely, whether it is favorable or unfavorable.
E. controllability.

198. Which of the following combinations of direct-material variances might prompt


management to undertake a detailed variance investigation?
A. Price, unfavorable; quantity, unfavorable.
B. Price, unfavorable; quantity, favorable.
C. Price, favorable; quantity, unfavorable.
D. Price favorable; quantity, favorable.
E. All of the above.

199. Consider the following statements about variance investigation:

I Variance investigation involves a look at only unfavorable variances.


II Variance investigation is typically based on a cost-benefit analysis.
III Variance investigation is often performed by establishing guidelines similar to the
following: Investigate variances that are greater than $X or greater than Y% of
standard cost.

Which of the above statements is (are) true?


A. I only.
B. II only.
C. III only.
D. II and III.
E. I, II, and III.
Chapter 14

200. A statistical control chart is best used for determining:


A. direct-material price variances.
B. direct-labor variances.
C. whether a variance is favorable or unfavorable.
D. who should be held accountable for specific variances.
E. whether a particular variance should be investigated.

201. The individual generally responsible for the direct-material price variance is the:
A. sales manager.
B. production supervisor.
C. purchasing manager.
D. finance manager.
E. head of the human resources department.

202. A production supervisor generally has little influence over the:


A. direct-material quantity variance.
B. direct-labor rate variance.
C. direct-labor efficiency variance.
D. direct-material price variance.
E. number of units produced.

203. In which department would an investigation normally begin regarding an unfavorable


materials quantity variance?
A. Quality control.
B. Purchasing.
C. Engineering.
D. Production.
E. Receiving.

204. Cohen Corporation has a favorable materials quantity variance. Which department
would likely be asked to explain the cause of this variance?
A. Engineering.
B. Purchasing.
C. Production.
D. Marketing.
E. None, because the variance is favorable.
Chapter 14

205. Rogers, Inc., had an unfavorable labor efficiency variance and an unfavorable materials
quantity variance. Which department might be held accountable for these variances?
A. Purchasing, because bad materials can harm labor efficiency.
B. Production, because inefficient workers may use more materials than allowed.
C. Purchasing and/or production.
D. Marketing.
E. Shipping.

206. A direct-material quantity variance can be caused by all of the following except:
A. improper employee training.
B. changes in sales volume.
C. acquisition of materials at a very attractive price.
D. adjustment problems with machines.
E. disgruntled workers.

207. A direct-labor efficiency variance cannot be caused by:


A. inexperienced employees.
B. poor quality raw materials.
C. employee inefficiency.
D. an out-of-date labor time standard.
E. producing fewer finished units than originally planned.

208. Justin Company recently purchased materials from a new supplier at a very attractive
price. The materials were found to be of poor quality, and the company's laborers
struggled significantly as they shaped the materials into finished product. In a
desperation move to make up for some of the time lost, the manufacturing supervisor
brought in more-senior employees from another part of the plant. Which of the following
variances would have a high probability of arising from this situation?
A. Material price variance, favorable.
B. Material quantity variance, unfavorable.
C. Labor rate variance, unfavorable.
D. Labor efficiency variance, unfavorable.
E. All of the above.
Chapter 14

209. Listed below are five variances (and possible causes) that are under review by
management of Knox Company. Which of the following is least likely to cause the variance
indicated?
A The need to ship goods acquired from a distant supplier via FedEx rather than via
truck; material price variance.
B The need to complete goods on a timely basis during a period of high absenteeism;
labor rate variance.
C A work-team that is very unhappy with its supervisor; labor efficiency variance.
D The need to close a plant for two days because of blizzard conditions; material
quantity variance, part no. 542.
E A malfunctioning piece of manufacturing equipment; labor efficiency variance.

210. Standard costs:


A. allow a manager to assess the efficiency of operations.
B. allow a company to practice management by exception.
C. provide management with a basis for performance evaluations.
D. if set correctly, can provide a motivational tool for employees.
E. will provide all of the above benefits for a company.

Answer: E LO: 7 Type: RC, N


Chapter 14

211. Which of the following is a criticism of standard costing, as applied to today's


manufacturing environment?
A. Automated manufacturing processes are very consistent in meeting production
specifications, making variances very small and relatively unimportant.
B. Variance information is usually aggregated (i.e., combined) rather than associated
with a particular batch of goods or a specific product line.
C. Traditional standard costing fails to focus on key business issues such as customer
service and bringing products to market faster than the competition.
D. Standard costing pays considerable attention to labor cost and labor efficiency,
which are becoming a relatively unimportant factor of production.
E. All of the above are valid criticisms.

212. Which of the following is not a valid way to adapt standard cost systems to today's
manufacturing environment?
A. Emphasize material and overhead costs.
B. Use more non-traditional cost drivers such as number of setups or number of
engineering change orders.
C. Update standards more frequently to adjust for the elimination of non-value-added
costs.
D. Use additional nonfinancial measures for performance evaluation and control.
E. Devote more resources to the tracking of direct labor cost.

213. To assess how customers perceive a company's products, management may study:
A. the number of customer complaints.
B. the number of warranty claims.
C. the number of products returned.
D. the cost of repairing returned products.
E. all of the above measures.

214. To improve its manufacturing efficiency, companies should strive toward increasing
__________ time as a percentage of processing time + inspection time + waiting time +
move time. The blank is:
A. processing time.
B. lead time.
C. waiting time.
D. move time.
E. inspection time.
Chapter 14

215. In the calculation of manufacturing cycle efficiency, which of the following activities
results in value-added time?
A. Moving.
B. Processing.
C. Inspection.
D. Waiting.
E. All of the above.

216. The manufacturing cycle efficiency for PQR Company when the processing time is six
hours and inspection, waiting, and move time are one hour each is:
A. 0.67.
B. 0.75.
C. 0.78.
D. 0.88.
E. an amount other than those shown above.

217. Which of the following would not be a concern of a company that desires to compete in a
global manufacturing arena?
A. Number of new products introduced.
B. Manufacturing cycle efficiency.
C. Number of customer complaints.
D. Number of on-time deliveries.
E. All of the above would be concerns.

218. An increasingly popular approach that integrates financial and customer performance
measures with measures in the areas of internal operations and learning and growth is
known as:
A. the integrated performance measurement tool (IPMT).
B. the balanced scorecard.
C. gain sharing.
D. cycle efficiency.
E. overall quality assessment (OQA).

219. The typical balanced scorecard is best described as containing:


A. financial performance measures.
B. nonfinancial performance measures.
C. neither financial nor nonfinancial performance measures.
D. both financial and nonfinancial performance measures.
E. both financial and nonfinancial performance measures, the latter often covering a
broad range of perspectives such as customers, internal operations, and learning
and growth.
Chapter 14

220. Swedish Cruise Lines (SCL), which operates in a very competitive marketplace, is
considering four categories of performance measures: (1) profitability measures, (2)
customer-satisfaction measures, (3) efficiency and quality measures, and (4) learning
and growth measures. The company assigns one manger to each ship in its fleet to
oversee the ship's general operations. If SCL desired to adopt a balanced-scorecard
approach, which measures should the firm use in the evaluation of its managers?
A. 1.
B. 1, 2.
C. 2, 3.
D. 1, 2, 4.
E. 1, 2, 3, 4.

221. Lead indicators guide management to:


A. take actions now that will have positive effects on organizational performance now.
B. take actions now that will have positive effects on organizational performance in the
future.
C. take actions in the future that will have positive effects on organizational
performance now.
D. take actions in the past that will have positive effects on organizational performance
in the future.
E. pursue identical strategies as those implemented with lag indicators.

222. When using a balanced scorecard, a company's market share is typically classified as
an element of the firm's:
A. financial performance measures.
B. customer performance measures.
C. learning and growth performance measures.
D. internal-operations performance measures.
E. interdisciplinary performance measures.

223. When using a balanced scorecard, which of the following is typically classified as an
internal-operations performance measure?
A. Cash flow.
B. Number of customer complaints.
C. Employee training hours.
D. Number of employee suggestions.
E. Number of suppliers used.

224. Which of the following perspectives is influenced by a company's vision and strategy?
A. Financial.
B. Customer.
C. Internal operations.
D. Learning and growth.
E. All of the above.

225. Which of the following journal entries definitely contains an error?


A. Raw-Material Inventory 200,000
Direct-Material Price Variance 5,000
Accounts Payable 205,000
Chapter 14

B. Raw-Material Inventory 38,000


Direct-Material Price Variance 2,000
Accounts Payable 36,000
C. Raw-Material Inventory 156,000
Direct-Material Price Variance 8,000
Work-in-Process Inventory 148,000
D. Work-in-Process Inventory 67,000
Direct-Material Quantity Variance 3,000
Raw-Material Inventory 70,000
E. Work-in-Process Inventory 79,000
Direct-Material Quantity Variance 4,000
Raw-Material Inventory 75,000

226. At the end of the accounting period, most companies close variance accounts to:
A. Raw-Material Inventory.
B. Work-in-Process Inventory.
C. Finished-Goods Inventory.
D. Cost of Goods Sold.
E. Income Summary.

227. Which of the following is NOT a type of absorption costing?


a. Direct costing.
b. Actual costing.
c. Normal costing.
d. None of the above.

228. Variable costing is UNACCEPTABLE for


a. managerial accounting.
b. financial accounting.
c. transfer pricing.
d. reporting by product lines for internal purposes.

229. A criticism of variable costing for managerial accounting purposes is that it


a. is not acceptable for product line segmented reporting.
b. does not reflect cost-volume-profit relationships.
c. overstates inventories.
d. might encourage managers to emphasize the short term at the expense of the
long term.

230. Normal costing and standard costing differ in that


a. the two systems can show different overhead budget variances.
b. only normal costing can be used with absorption costing.
c. the two systems show different volume variances if standard hours do not
equal actual hours.
d. normal costing is less appropriate for multiproduct firms.

231. Variable costing and absorption costing will show the same incomes when there
Chapter 14

are no
a. beginning inventories.
b. ending inventories.
c. variable costs.
d. beginning and ending inventories.

232. ABC had the same activity in 20X3 as in 20X2 except that production was higher in
20X3 than in 20X2. ABC will show
a. higher income in 20X3 than in 20X2.
b. the same income in both years.
c. the same income in both years under variable costing.
d. the same income in both years under absorption costing.

233. The use of variable costing requires knowing


a. the contribution margin and break-even point for each product.
b. the variable and fixed components of production cost.
c. controllable and noncontrollable components of all costs.
d. the number of units of each product produced during the period.
Chapter 14

234. Which measure of activity is likely to give the LOWEST standard fixed cost per
unit?
a. Actual activity.
b. Normal capacity.
c. Budgeted activity.
d. Practical capacity.

235. Which item is NOT used to compute the fixed overhead volume variance?
a. Standard fixed cost per unit.
b. Budgeted fixed overhead.
c. Actual fixed overhead.
d. Actual quantity produced.

236. Which variance is LEAST relevant for control purposes?


a. Material use variance.
b. Fixed overhead volume variance.
c. Fixed overhead budget variance.
d. Labor efficiency variance.

237. A company that sets a standard fixed cost based on practical capacity
a. should expect unfavorable volume variances.
b. will set its selling prices too low.
c. has a higher cost per unit than a company using normal activity to set the
standard.
d. usually overapplies its fixed costs.

238. A predetermined overhead rate for fixed costs is unlike a standard fixed cost per
unit in that a predetermined overhead rate is
a. based on an input factor like direct labor hours and a standard cost per unit is
based on a unit of output.
b. based on practical capacity and a standard fixed cost can be based on any
level of activity.
c. used with variable costing while a standard fixed cost is used with absorption
costing.
d. likely to be higher than a standard fixed cost per unit.

239. Advocates of variable costing for internal reporting purposes do NOT rely on which
of the following points?
a. The matching concept.
b. Price-volume relationships.
c. Absorption costing does not include selling and administrative expenses as
part of inventoriable cost.
d. Production influences income under absorption costing.
Chapter 14

240. Calculating income under variable costing does NOT require knowing
a. unit sales.
b. unit variable manufacturing costs.
c. selling price.
d. unit production.

241. Inventoriable costs under absorption costing include


a. both fixed and variable production costs.
b. only variable production costs.
c. all production costs plus variable selling and administrative costs.
d. all production costs plus all selling and administrative costs.

242. Inventoriable costs under variable costing include


a. fixed and variable production costs.
b. variable production costs.
c. all production costs plus variable selling and administrative costs.
d. all production costs plus all selling and administrative costs.

243. Absorption costing and variable costing differ in that


a. income is lower under variable costing.
b. variable costing treats selling costs as period costs.
c. variable costing treats all variable costs as product costs.
d. inventory cost is higher under absorption costing.

244. Absorption costing differs from variable costing in that


a. standards can be used with absorption costing, but not with variable costing.
b. absorption costing inventories are more correctly valued.
c. production influences income under absorption costing, but not under variable
costing.
d. companies using absorption costing have lower fixed costs.

245. Which method gives the lowest inventory cost per unit?
a. Variable costing.
b. Absorption costing using normal activity to set the standard fixed cost.
c. Absorption costing using practical capacity to set the standard fixed cost.
d. Actual absorption costing.

246. Which costs are treated differently under absorption costing and variable costing?
a. Variable manufacturing costs.
b. Fixed manufacturing costs.
c. Variable selling and administrative expenses.
d. Fixed selling and administrative expenses.

247. ABC Company had 15,000 units in ending inventory. The total cost of those units
under variable costing is
a. less than it is under absorption costing.
Chapter 14

b. the same as it is under absorption costing.


c. more than it is under absorption costing.
d. any of the above.

248. An unfavorable volume variance means that


a. cost control was probably poor.
b. absorption costing income is lower than variable costing income.
c. actual output was less than the level used to set the standard fixed cost.
d. actual output was more than the level used to set the standard fixed cost.

249. Which variance CANNOT arise under variable costing?


a. variable overhead budget variance.
b. variable overhead efficiency variance.
c. fixed overhead budget variance.
d. fixed overhead volume variance.

250. Standard costing differs from normal costing in the treatment of


a. materials, direct labor, and overhead.
b. materials and direct labor.
c. direct labor and overhead.
d. overhead.

251. Normal costing differs from actual costing in treating


a. materials, direct labor, and overhead.
b. materials and direct labor.
c. direct labor and overhead.
d. overhead.

252. As compared to normal costing, standard costing can yield


a. different volume variances and budget variances.
b. different budget variances.
c. different volume variances.
d. none of the above.

253. Under variable costing there can be no


a. fixed overhead variances.
b. fixed overhead budget variance.
c. fixed overhead volume variance.
d. no fixed overhead.

254. ABC had the same activity in 20X4 as in 20X3 except that production was lower in
20X4 than in 20X3. ABC will show
a. lower income in 20X4 than in 20X3.
b. the same income in both years.
c. the same income in both years under variable costing.
d. the same income in both years under absorption costing.
Chapter 14

255. What is a standard cost?


a. The total number of units times the budgeted amount expected
b. Any amount that appears on a budget
c. The total amount that appears on the budget for product costs
d. The amount management thinks should be incurred to produce a good or service

256. A standard cost is


a. a cost which is paid for a group of similar products.
b. the average cost in an industry.
c. a predetermined cost.
d. the historical cost of producing a product last year.

257. The difference between a budget and a standard is that


a. a budget expresses what costs were, while a standard expresses what costs should
be.
b. a budget expresses management's plans, while a standard reflects what actually
happened.
c. a budget expresses a total amount, while a standard expresses a unit amount.
d. standards are excluded from the cost accounting system, whereas budgets are
generally incorporated into the cost accounting system.

258. Standard costs may be used by


a. universities.
b. governmental agencies.
c. charitable organizations.
d. all of these.

259. Which of the following statements is false?


a. A standard cost is more accurate than a budgeted cost.
b. A standard is a unit amount.
c. In concept, standards and budgets are essentially the same.
d. The standard cost of a product is equivalent to the budgeted cost per unit of product.

260. Budget data are not journalized in cost accounting systems with the exception of
a. the application of manufacturing overhead.
b. direct labor budgets.
c. direct materials budgets.
d. cash budget data.

261. It is possible that a company's financial statements may report inventories at


a. budgeted costs.
b. standard costs.
c. both budgeted and standard costs.
d. none of these.

262. A standard differs from a budget because a standard


a. is a predetermined cost.
b. contributes to management planning and control.
Chapter 14

c. is a unit amount.
d. none of the above; a standard does not differ from a budget.
263. Using standard costs
a. makes employees less “cost-conscious.”
b. provides a basis for evaluating cost control.
c. makes management by exception more difficult.
d. increases clerical costs.

264. Using standard costs


a. can make management planning more difficult.
b. promotes greater economy.
c. does not help in setting prices.
d. weakens management control.

265. If standard costs are incorporated into the accounting system,


a. it may simplify the costing of inventories and reduce clerical costs.
b. it can eliminate the need for the budgeting process.
c. the accounting system will produce information that is less relevant than the
historical cost accounting system.
d. approval of the stockholders is required.

266. Standard costs


a. may show past cost experience.
b. help establish expected future costs.
c. are the budgeted cost per unit in the present.
d. all of these.

267. Which of the following statements about standard costs is false?


a. Properly set standards should promote efficiency.
b. Standard costs facilitate management planning.
c. Standards should not be used in "management by exception."
d. Standard costs can simplify the costing of inventories.

268. Which of the following is not considered an advantage of using standard costs?
a. Standard costs can reduce clerical costs.
b. Standard costs can be useful in setting prices for finished goods.
c. Standard costs can be used as a means of finding fault with performance.
d. Standard costs can make employees "cost-conscious."

269. If a company is concerned with the potential negative effects of establishing


standards, it should
a. set loose standards that are easy to fulfill.
b. offer wage incentives to those meeting standards.
c. not employ any standards.
d. set tight standards in order to motivate people.

270. The two levels that standards may be set at are


a. normal and fully efficient.
b. normal and ideal.
Chapter 14

c. ideal and less efficient.


d. fully efficient and fully effective.

271. The most rigorous of all standards is the


a. normal standard.
b. realistic standard.
c. ideal standard.
d. conceivable standard.

272. Most companies that use standards set them at


a. the normal level.
b. a conceivable level.
c. the ideal level.
d. last year's level.

273. A managerial accountant


1. does not participate in the standard setting process.
2. provides knowledge of cost behaviors in the standard setting process.
3. provides input of historical costs to the standard setting process.
a. 1
b. 2
c. 3
d. 2 and 3

274. The cost of freight-in


a. is to be included in the standard cost of direct materials.
b. is considered a selling expense.
c. should have a separate standard apart from direct materials.
d. should not be included in a standard cost system.

275. The direct materials quantity standard would not be expressed in


a. pounds.
b. barrels.
c. dollars.
d. board feet.

276. The direct materials quantity standard should


a. exclude unavoidable waste.
b. exclude quality considerations.
c. allow for normal spoilage.
d. always be expressed as an ideal standard.

277. The direct labor quantity standard is sometimes called the direct labor
a. volume standard.
b. effectiveness standard.
c. efficiency standard.
d. quality standard.
Chapter 14

278. A manufacturing company would include setup and downtime in their direct
a. materials price standard.
b. materials quantity standard.
c. labor price standard.
d. labor quantity standard.

279. Allowance for spoilage is part of the direct


a. materials price standard.
b. materials quantity standard.
c. labor price standard.
d. labor quantity standard.

280. The total standard cost to produce one unit of product is shown
a. at the bottom of the income statement.
b. at the bottom of the balance sheet.
c. on the standard cost card.
d. in the Work in Process Inventory account.

281. An unfavorable materials quantity variance would occur if


a. more materials were purchased than were used.
b. actual pounds of materials used were less than the standard pounds allowed.
c. actual labor hours used were greater than the standard labor hours allowed.
d. actual pounds of materials used were greater than the standard pounds allowed.

282. A standard which represents an efficient level of performance that is attainable


under expected operating conditions is called a(n)
a. ideal standard.
b. loose standard.
c. tight standard.
d. normal standard.

283. Ideal standards


a. are rigorous but attainable.
b. are the standards generally used in a master budget.
c. reflect optimal performance under perfect operating conditions.
d. will always motivate employees to achieve the maximum output.

284. The final decision as to what standard costs should be is the responsibility of
a. the quality control engineer.
b. the managerial accountants.
c. the purchasing agent.
d. management.

285. The labor time requirements for standards may be determined by the
a. sales manager.
b. product manager.
c. industrial engineers.
d. payroll department manager.
Chapter 14

286. To determine the standard rate for direct labor, management consults
a. purchasing agents.
b. product managers.
c. quality control engineers.
d. the payroll department.
287. The standard direct materials quantity does not include allowances for
a. unavoidable waste.
b. normal spoilage.
c. unexpected spoilage.
d. all of the above are included.

288. Allowances should not be made in the direct labor quantity standard for
a. wasted time.
b. rest periods.
c. cleanup.
d. machine downtime.

289. The standard predetermined overhead rate used in setting the standard
overhead cost is determined by dividing
a. budgeted overhead costs by an expected standard activity index.
b. actual overhead costs by an expected standard activity index.
c. budgeted overhead costs by actual activity.
d. actual overhead costs by actual activity.
290. Which of the following statements is true?
a. Variances are the differences between total actual costs and total standard costs.
b. When actual costs exceed standard costs, the variance is favorable.
c. An unfavorable variance results when actual costs are decreasing but standards are
not changed.
d. All of the above are true.

291. The matrix approach to variance analysis


a. will yield slightly different variances than the formula approach.
b. is more accurate than the formula approach.
c. does not separate the price and quantity variance calculations.
d. provides a convenient structure for determining each variance.

292. Labor efficiency is measured by the


a. materials quantity variance.
b. total labor variance.
c. labor quantity variance.
d. labor rate variance.

293. An unfavorable labor quantity variance may be caused by


a. paying workers higher wages than expected.
b. misallocation of workers.
c. worker fatigue or carelessness.
d. higher pay rates mandated by union contracts.

294. The investigation of materials price variance usually begins in the


Chapter 14

a. first production department.


b. purchasing department.
c. controller's office.
d. accounts payable department.

295. The investigation of a materials quantity variance usually begins in the


a. production department.
b. purchasing department.
c. sales department.
d. controller's department.

296. If the labor quantity variance is unfavorable and the cause is inefficient use of
direct labor, the responsibility rests with the
a. sales department.
b. production department.
c. budget office.
d. controller's department.

297. Which one of the following describes the total overhead variance?
a. The difference between what was actually incurred and the flexible budget amount
b. The difference between what was actually incurred and overhead applied
c. The difference between the overhead applied and the flexible budget amount
d. The difference between what was actually incurred and the total production budget
298. If actual direct materials costs are greater than standard direct materials costs, it
means that
a. actual costs were calculated incorrectly.
b. the actual unit price of direct materials was greater than the standard unit price of
direct materials.
c. the actual unit price of raw materials or the actual quantities of raw materials used
was greater than the standard unit price or standard quantities of raw materials
expected.
d. the purchasing agent or the production foreman is inefficient.

299. If actual costs are greater than standard costs, there is a(n)
a. normal variance.
b. unfavorable variance.
c. favorable variance.
d. error in the accounting system.

300. A total materials variance is analyzed in terms of


a. price and quantity variances.
b. buy and sell variances.
c. quantity and quality variances.
d. tight and loose variances.
301. The purchasing agent of the Skateboard Company ordered materials of lower
quality in an effort to economize on price. What variance will most likely result?
a. Favorable materials quantity variance
b. Favorable total materials variance
c. Unfavorable materials price variance
Chapter 14

d. Unfavorable labor quantity variance


302. Which one of the following statements is true?
a. If the materials price variance is unfavorable, then the materials quantity variance
must also be unfavorable.
b. If the materials price variance is unfavorable, then the materials quantity variance
must be favorable.
c. Price and quantity variances move in the same direction. If one is favorable, the
others will be as well.
d. There is no correlation of favorable or unfavorable for price and quantity variances.

303. Variances from standards are


a. expressed in total dollars.
b. expressed on a per-unit basis.
c. expressed on a percentage basis.
d. all of these.

304. A favorable variance


a. is an indication that the company is not operating in an optimal manner.
b. implies a positive result if quality control standards are met.
c. implies a positive result if standards are flexible.
d. means that standards are too loosely specified.

305. The total materials variance is equal to the


a. materials price variance.
b. difference between the materials price variance and materials quantity variance.
c. product of the materials price variance and the materials quantity variance.
d. sum of the materials price variance and the materials quantity variance.

306. The total overhead variance is equal to the


a. sum of the total materials variance and the total labor variance.
b. difference between the total materials variance and the total labor variance.
c. sum of the controllable variance and the volume variance.
d. total variance minus the controllable variance and the volume variance.
307. Unfavorable materials price and quantity variances are generally the
responsibility of the
Price Quantity
a. Purchasing department Purchasing Department
b. Purchasing department Production Department
c. Production department Production Department
d. Production Department Purchasing Department
Chapter 14

308. The total overhead variance is the difference between the


a. actual overhead costs and overhead costs applied based on standard hours allowed.
b. actual overhead costs and overhead costs applied based on actual hours.
c. overhead costs applied based on actual hours and overhead costs applied based on
standard hours allowed.
d. the actual overhead costs and the standard direct labor costs.
309. Which of the following is true?
a. The form, content, and frequency of variance reports vary considerably among
companies.
b. The form, content, and frequency of variance reports do not vary among companies.
c. The form and content of variance reports vary considerably among companies, but
the frequency is always weekly.
d. The form and content of variance reports are consistent among companies, but the
frequency varies.
310. When is a variance considered to be 'material'?
a. When it is large compared to the actual cost
b. When it is infrequent
c. When it is unfavorable
d. When it could have been controlled more effectively
311. Variance reports are
a. external financial reports.
b. SEC financial reports.
c. internal reports for management.
d. all of these.

322. In using variance reports, management looks for


a. total assets invested.
b. significant variances.
c. competitors’ costs in comparison to the company's costs.
d. more efficient ways of valuing inventories.

323. Magliano Company prepared its income statement for internal use. How would
amounts for cost of goods sold and variances appear?
a. Cost of goods sold would be at actual costs, and variances would be reported
separately.
b. Cost of goods sold would be combined with the variances, and the net amount
reported at standard cost.
c. Cost of goods sold would be at standard costs, and variances would be reported
separately.
d. Cost of goods sold would be combined with the variances, and the net amount
reported at actual cost.

324. Dell Widgets prepared its income statement for management using a standard
cost accounting system. Which of the following appears at the “standard”
amount?
a. Sales
b. Selling expenses
c. Gross profit
d. Cost of goods sold
Chapter 14

325. The costing of inventories at standard cost for external financial statement
reporting purposes is
a. not permitted.
b. preferable to reporting at actual costs.
c. in accordance with generally accepted accounting principles if significant differences
exist between actual and standard costs.
d. in accordance with generally accepted accounting principles if significant differences
do not exist between actual and standard costs.

326. Income statements prepared internally for management often show cost of goods
sold at standard cost and variances are
a. separately disclosed.
b. deducted as other expenses and revenues.
c. added to cost of goods sold.
d. closed directly to retained earnings.
327. The balanced scorecard
a. incorporates financial and nonfinancial measures in an integrated system.
b. is based on financial measures.
c. is based on nonfinancial measures.
d. does not use financial or nonfinancial neasures.

328. Which is not one of the four most commonly used perspectives on a balanced
scorecard?
a. The financial perspective
b. The customer perspective
c. The external process perspective
d. The learning and growth perspective

329. The balanced scorecard approach


a. uses only financial measures to evaluate performance.
b. uses rather vague, open statements when setting objectives in order to allow
managers and employees flexibility.
c. normally sets the financial objectives first, and then sets the objectives in the other
perspectives to accomplish the financial objectives.
d. evaluates performance using about 10 different perspectives in order to effectively
incorporate all areas of the organization.
Chapter 14

330. The customer perspective of the balanced scorecard approach


a. is the most traditional view of the company.
b. evaluates the internal operating processes critical to the success of the organization.
c. evaluates how well the company develops and retains its employees.
d. evaluates how well the company is performing from the viewpoint of those people
who buy its products and services.

331. The perspectives included in the balanced scorecard approach include all of the
following except the
a. internal process perspective.
b. capacity utilization perspective.
c. learning and growth perspective.
d. customer perspective.
a
332. Debit balances in variance accounts represent
a. unfavorable variances.
b. favorable variances.
c. favorable for price variances; unfavorable for quantity variances.
d. favorable for quantity variances; unfavorable for price variances.

a
333. Manufacturing overhead costs are applied to work in process on the basis of
a. actual hours worked.
b. standard hours allowed.
c. ratio of actual variable to fixed costs.
d. actual overhead costs incurred.

a
334. The overhead variances measure whether overhead costs
Are Effectively Managed Were Used Effectively
a. Controllable Controllable and Volume
b. Controllable Volume
c. Controllable and Volume Controllable
d. Volume Controllable

a
335. The overhead volume variance is
a. actual overhead less overhead budgeted for actual hours.
b. actual overhead less overhead budgeted for standard hours allowed.
c. overhead budgeted for actual hours less applied overhead.
d. the fixed overhead rate times the difference between normal capacity hours and
standard hours allowed.

a
336. Which of the following statements is false?
a. The overhead volume variance indicates whether plant facilities were used efficiently
during the period.
b. The costs that cause the overhead volume variance are usually controllable costs.
c. The overhead volume variance relates solely to fixed costs.
d. The overhead volume variance is favorable if standard hours allowed for output are
greater than the standard hours at normal capacity.
Chapter 14

a
337. If the standard hours allowed are less than the standard hours at normal
capacity,
a. the overhead volume variance will be unfavorable.
b. variable overhead costs will be underapplied.
c. the overhead controllable variance will be favorable.
d. variable overhead costs will be overapplied.
a
338. Which of the following statements about overhead variances is false?
a. Standard hours allowed are used in calculating the controllable variance.
b. Standard hours allowed are used in calculating the volume variance.
c. The controllable variance pertains solely to fixed costs.
d. The total overhead variance pertains to both variable and fixed costs.

a
339. The overhead volume variance relates only to
a. variable overhead costs.
b. fixed overhead costs.
c. both variable and fixed overhead costs.
d. all manufacturing costs.

340. What does the controllable variance measure?


a. Whether a company incurred more or less fixed overhead costs compared to the
amount of overhead applied
b. Whether a company incurred more or less overhead costs than allowed
c. The efficiency of using variable overhead resources
d. Whether the production manager is able to control the production facility

a
341. The overhead controllable variance is calculated as the difference between
actual overhead costs incurred and the budgeted
a. overhead costs for the standard hours allowed.
b. overhead costs applied to the product.
c. overhead costs at the normal level of activity.
d. fixed overhead costs.

a
342. If the standard hours allowed are less than the standard hours at normal
capacity, the volume variance
a. cannot be calculated.
b. will be favorable.
c. will be unfavorable.
d. will be greater than the controllable variance.

a
343. The budgeted overhead costs for standard hours allowed and the overhead costs
applied to the product are the same amount
a. for both variable and fixed overhead costs.
b. only when standard hours allowed are less than normal capacity.
c. for variable overhead costs.
d. for fixed overhead costs.
a
344. An overhead volume variance is calculated as the difference between normal
capacity hours and standard hours allowed
a. times the total predetermined overhead rate.
Chapter 14

b. times the predetermined variable overhead rate.


c. times the predetermined fixed overhead rate.
d. divided by actual number of hours worked.

345. All of the following are advantages of standard costs except they
a. facilitate management planning.
b. are useful in setting selling prices.
c. simplify costing in inventories.
d. increase net income.

346. Standards based on the optimum level of performance under perfect operating
conditions are
a. attainable standards.
b. ideal standards.
c. normal standards.
d. practical standards.

347. The direct materials price standard should include an amount for all of the
following except
a. receiving costs.
b. storing costs.
c. handling costs.
d. normal spoilage costs.

348. The standard unit cost is used in the calculation of which of the following
variances?
Materials Price Variance Materials Quantity Variance
a. No No
b. No Yes
c. Yes No
d. Yes Yes

349. The difference between the actual labor rate multiplied by the actual labor hours
worked and the standard labor rate multiplied by the standard labor hours is the
a. total labor variance.
b. labor price variance.
c. labor quantity variance.
d. labor efficiency variance.

350. Which department is usually responsible for a labor price variance attributable to
misallocation of workers?
a. Quality control
b. Purchasing
c. Engineering
d. Production

351. In reporting variances,


a. promptness is relatively unimportant.
Chapter 14

b. management normally investigates all variances.


c. the reports should facilitate management by exception.
d. the reports are not departmentalized.

352. A standard cost system may be used in


Job Order Costing Process Costing
a. No No
b. Yes No
c. No Yes
d. Yes Yes
353. To measure controllable production inefficiencies, which of the following is the best
basis for a company to use in establishing the standard hours allowed for the
output of one unit of product?
A) Average historical performance for the last several years.
B) Engineering estimates based on ideal performance.
C) Engineering estimates based on attainable performance.
D) The hours per unit that would be required for the present workforce to
satisfy expected demand over the long run.

354.Poorly trained workers could have an unfavorable effect on which of the following
variances?

Labor Rate Variance Materials Quantity Variance


A) Yes Yes
B) Yes No
C
) No Yes
D
) No No

355. Richter Corp. recorded the following entry in its general ledger:

6,00
Work in Process 0
Material Quantity Variance 500
6,50
Raw Materials 0

The above journal entry indicates that:


A) the materials quantity variance for the period was favorable.
B) less materials were used in production during the period than was called for
at standard.
C) the materials quantity variance for the period was unfavorable.
D) the actual price paid for the materials used in production was greater than
the standard price allowed.
Chapter 14

356. When the actual price paid on credit for a raw material exceeds its standard
price, the journal entry would include:
A) Credit to Raw Materials; Credit to Materials Price Variance
B) Credit to Accounts Payable; Credit to Materials Price Variance
C) Credit to Raw Materials; Debit to Materials Price Variance
D) Credit to Accounts Payable; Debit to Materials Price Variance

357. The variance that is most useful in assessing the performance of the purchasing
department manager is:
A) the materials quantity variance.
B) the materials price variance.
C) the labor rate variance.
D) the labor efficiency variance.

358. The production department should generally be responsible for material price
variances that resulted from:
A) purchases made in uneconomical lot-sizes.
B) rush orders arising from poor scheduling.
C) purchase of the wrong grade of materials.
D) changes in the market prices of raw materials.

359. A debit balance in the labor efficiency variance account indicates that:
A) standard hours exceed actual hours.
B) actual hours exceed standard hours.
C) standard rate and standard hours exceed actual rate and actual hours.
D) actual rate and actual hours exceed standard rate and standard hours.

360. The journal entry below:

25,00
Work in Process 0
Direct Labor Efficiency
Variance 1,200
Direct Labor Rate
Variance 2,000
24,20
Accrued Wages Payable 0
Chapter 14

indicates that:
A) the total labor variance was $800, unfavorable.
B) employees received an unexpected rate increase during the period.
C) more labor time was required to complete the output of the period than was
allowed at standard.
D) responses a and b are both correct.

361. When the actual wage rate paid to direct labor workers exceeds the standard
wage rate, the journal entry would include:
A) Debit to Wages Payable; Credit to Labor Rate Variance
B) Debit to Work-In-Process; Credit to Labor Rate Variance
C) Debit to Wages Payable; Debit to Labor Rate Variance
D) Debit to Work-In-Process; Debit to Labor Rate Variance

362. During a recent lengthy strike at Morell Manufacturing Company, management


replaced striking assembly line workers with office workers. The assembly line
workers were being paid $18 per hour while the office workers are only paid $10
per hour. What is the most likely effect on the labor variances in the first month of
this strike?

Labor Rate Variance Labor Efficiency Variance


A) Unfavorable No effect
B) No effect Unfavorable
C
) Unfavorable Favorable
D
) Favorable Unfavorable

363. Which of the following will increase a company's manufacturing cycle efficiency
(MCE)?

Decrease in Process Time Decrease in Wait Time


A) Yes Yes
B) Yes No
C
) No Yes
D
) No No
Chapter 14

364. Which of the following conditions normally would not indicate that standard costs should be
revised?
a. The engineering department has revised product specifications in responding to
customer suggestions.
b. The company has signed a new union contract which increases the factory wages on
average by $2.00 an hour.
c. Actual costs differed from standard costs for the preceding week.
d. The world price of raw materials increased.

365.Standards that represent levels of operation that can be attained with reasonable effort are
called:
a. theoretical standards
b. ideal standards
c. variable standards
d. normal standards
366. Manufacturing companies use standard costs for the following except:
a. Variable costs
b. Direct Materials
c. Direct Labor
d. Factory Overhead

367.Standard costs are used in companies for a variety of reasons. Which of the following is not
one of the benefits for using standard costs?
a. Used to indicate where changes in technology and machinery need to be made.
b. Used to value inventory
c. Used to plan direct materials, direct labor, and factory manufacturing cost.
d. Used to control costs.

368.The principle of exceptions allows managers to


a. focus on correcting variances between standard costs and actual costs.
b. focus on correcting variances between variable costs and actual costs.
c. focus on correcting variances between competitor’s costs and actual costs.
d. focus on correcting variances between competitor’s costs and standard costs.

369.Several people play an essential part in setting standards. Which of the following is
incorrect as to setting standards?
a. Accountants expresses judgement in dollars and cents.
b. Engineers identify material, labor, and machine requirements.
c. Human resource managers provide personnel information.
d. Quality managers provide quality measures that will be used to evaluate rejects.

370.Periodic comparisons between planned objectives and actual performance are reported in:
a. zero-base reports
b. budget performance reports
c. master budgets
d. budgets

371.The standard price and quantity of direct materials are separated because:
a. GAAP reporting requires this separation
Chapter 14

b. direct materials prices are controlled by the purchasing department, and quantity used is
controlled by the production department
c. standard quantities are more difficult to estimate than standard prices
d. standard prices change more frequently than standard quantities

372.Standard costs are divided into which of the following components?


a. Price Standard and Quantity Standard
b. Materials Standard and Labor Standard
c. Quality Standard and Quantity Standard
d. Price Standard and Quantity Standard
373.A favorable cost variance occurs when
a. Actual costs are more than standard costs.
b. Standard costs are more than actual costs.
c. Standard costs are less than actual costs.
d. None of the above.

374.Total manufacturing cost variance includes:


a. Direct materials price variance, direct labor cost variance, and fixed factory overhead
volume variance
b. Direct materials cost variance, direct labor rate variance, and factory overhead cost
variance
c. Direct materials cost variance, direct labor cost variance, variable factory overhead
controllable variance
d. Direct materials cost variance, direct labor cost variance, factory overhead cost variance

375.Which of the following is not a reason standard costs are separated in two components?
a. the price and quantity variances need to be identified separately to correct the actual
major differences.
b. identifying variances determines which manager must find a solution to major
discrepancies.
c. if a negative variance is over-shadowed by a favorable variance, managers may
overlook potential corrections.
d. variances brings attention to discrepancies in the budget and requires managers to
revise budgets closer to actual.
Chapter 14

376.If the actual quantity of direct materials used in producing a commodity differs from the
standard quantity, the variance is termed:
a. controllable variance
b. price variance
c. quantity variance
d. rate variance

377.If the price paid per unit differs from the standard price per unit for direct materials, the
variance is termed:
a. variable variance
b. controllable variance
c. price variance
d. volume variance

378.If the wage rate paid per hour differs from the standard wage rate per hour for direct labor,
the variance is termed:
a. variable variance
b. rate variance
c. quantity variance
d. volume variance

379.If the actual direct labor hours spent producing a commodity differ from the standard hours,
the variance is termed:
a. time variance
b. price variance
c. quantity variance
d. rate variance

380. Which of the following is not a reason for a direct materials quantity variance?
a. Malfunctioning equipment
b. Purchasing of inferior raw materials
c. Material requiring rework
d. Spoilage of materials

381.The formula to compute direct labor rate variance is to calculate the difference between
a. actual costs + (actual hours * standard rate)
b. actual costs - standard cost
c. (actual hours * standard rate) - standard costs
d. actual costs - (actual hours * standard rate)

382.The formula to compute direct labor time variance is to calculate the difference between
a. actual costs - standard costs
b. actual costs + standard costs
c. (actual hours * standard rate) - standard costs
d. actual costs - (actual hours * standard rate)

383.The formula to compute direct materials price variance is to calculate the difference
between
a. actual costs - (actual quantity * standard price)
b. actual cost + standard costs
Chapter 14

c. actual cost - standard costs


d. (actual quantity * standard price) -standard costs

384.The formula to compute direct material quantity variance is to calculate the difference
between
a. actual costs - standard costs
b. standard costs - actual costs
c. (actual quantity * standard price) - standard costs
d. actual costs - (standard price * standard costs)

385.Which of the following would not lend itself to applying direct labor variances?
a. Help desk
b. Administrative assistant
c. Customer service personnel
d. Telemarketer

386.Assuming that the standard fixed overhead rate is based on full capacity, the cost of
available but unused productive capacity is indicated by the:
a. factory overhead cost volume variance
b. direct labor cost time variance
c. direct labor cost rate variance
d. factory overhead cost controllable variance
387.Incurring actual indirect factory wages in excess of budgeted amounts for actual production
results in a:
a. quantity variance
b. controllable variance
c. volume variance
d. rate variance

388.The controllable variance measures:


a. operating results at less than normal capacity
b. the efficiency of using variable overhead resources
c. operating results at more than normal capacity
d. control over fixed overhead costs

389.The unfavorable volume variance may be due to all but the following factors:
a. failure to maintain an even flow of work
b. machine breakdowns
c. unexpected increases in the cost of utilities
d. failure to obtain enough sales orders

390.Favorable volume variances may be harmful when:


a. machine repairs cause work stoppages
b. supervisors fail to maintain an even flow of work
c. production in excess of normal capacity cannot be sold
d. there are insufficient sales orders to keep the factory operating at normal capacity
Chapter 14

391.A negative fixed overhead volume variance can be caused due to the following except:
a. Sales orders at a low level
b. Machine breakdowns
c. Employee inexperience
d. Increase in utility costs
392.At the end of the fiscal year, variances from standard costs are usually transferred to the:
a. direct labor account
b. factory overhead account
c. cost of goods sold account
d. direct materials account

393.Variances from standard costs are usually reported to:


a. suppliers
b. stockholders
c. management
d. creditors

394.If at the end of the fiscal year the variances from standard are significant, the variances
should be transferred to the:
a. work in process account
b. cost of goods sold account
c. finished goods account
d. work in process, cost of goods sold, and finished goods accounts

395.The use of standards for nonmanufacturing expenses is:


a. not as common as it is for manufacturing costs
b. as common as it is for manufacturing costs
c. not useful
d. impossible
Standard cost system
396. A primary purpose of using a standard cost system is
A. To make things easier for managers in the production facility.
B. To provide a distinct measure of cost control.
C. To minimize the cost per unit of production.
D. b and c are correct

397. Which one of the following statements is true concerning standard costs?
A. Standard costs are estimates of costs attainable only under the most ideal conditions,
but rarely practicable.
B. Standard costs are difficult to use with a process-costing system.
C. If properly used, standards can help motivate employees.
D. Unfavorable variances, material in amount, should be investigated, but large favorable
variances need not be investigated.

398. Which of the following is a purpose of standard costing?


A. Determine “breakeven” production level
B. Control costs
C. Eliminate the need for subjective decisions by management
D. Allocate cost with more accuracy
Chapter 14

399. When evaluating the operating performance management sometimes uses the
difference between expected and actual performance. This refers to:
A. Management by Deviation C. Management by Objective
B. Management by Control D. Management by Exception

400. The best basis upon which cost standards should be set to measure controllable
production inefficiencies is
A. Engineering standards based on ideal performance
B. Normal capacity
C. Engineering standards based on attainable performance
D. Practical capacity

401. A company employing very tight (high) standards in a standard cost system should
expect that
A. No incentive bonus will be paid.
B. Most variances will be unfavorable.
C. Employees will be strongly motivated to attain the standard.
D. Costs will be controlled better than if lower standards were used.

402. To measure controllable production inefficiencies, which of the following is the best basis
for a company to use in establishing the standard hours allowed for the output of one unit of
product?
A. Average historical performance for the last several years
B. Engineering estimates based on ideal performance
C. Engineering estimates based on attainable performance
D. The hours per unit that would be required for the present workforce to satisfy expected
demand over the long run

403. Which of the following statements about the selection of standards is true?
A. Ideal standards tend to extract higher performance levels since they give employees
something to live up to.
B. Currently attainable standards may encourage operating inefficiencies.
C. Currently attainable standards discourage employees from achieving their full
performance potential.
D. Ideal standards demand maximum efficiency which may leave workers frustrated, thus
causing a decline in performance.

404. A difference between standard costs used for cost control and the budgeted costs
representing the same manufacturing effort can exist because
A. standard costs must be determined after the budget is completed
B. standard costs represent what costs should be while budgeted costs represent expected
actual costs
C. budgeted costs are historical costs while standard costs are based on engineering
studies
D. budgeted costs include some “slack” or “padding” while standard costs do not

405. When standard costs are used in a process-costing system, how, if at all, are equivalent
units involved or used in the cost report at standard?
A. Equivalent units are not used.
B. Equivalent units are computed using a “special” approach.
Chapter 14

C. The actual equivalent units are multiplied by the standard cost per unit.
D. The standard equivalent units are multiplied by the actual cost per unit.

406. The fixed overhead application rate is a function of a predetermined “normal” activity
level. If standard hours allowed for good output equal this predetermined activity level for a
given period, the volume variance will be
A. Zero
B. Favorable
C. Unfavorable
D. Either favorable or unfavorable, depending on the budgeted overhead.

407. The absolute minimum cost possible under the best conceivable operating conditions is
a description of which type of standard?
A. Currently attainable (expected) C. Theoretical
B. Normal D. Practical

408. Standards, which are difficult to achieve due to reasons beyond the individual performing
the task, are the result of firm using which of the following methods to establish standards?
A. Ideal Standards C. Practical Standards
B. Lax Standards D. Employee Standards

409. Standards that represent levels of operation that can be attained with reasonable effort
are called:
A. Theoretical standards C. Variable standards
B. Ideal standards D. Normal standards

410. When performing input/output variance analysis in standard costing, “standard hours
allowed” is a means of measuring
A. Standard output at standard hours C. Actual output at standard hours
B. Standard output at actual hours D. Actual output at actual hours

411. A company uses a two-way analysis for overhead variances: budget (controllable) and
volume. The volume variance is based on the
A. Total overhead application rate
B. Volume of total expenses at various activity levels
C. Variable overhead application rate
D. Fixed overhead application rate

412. Assuming that the standard fixed overhead rate is based on full capacity, the cost of
available but unused productive capacity is indicated by the:
A. Factory overhead cost volume variance
B. Direct labor cost efficiency variance
C. Direct labor cost rate variance
D. Factory overhead cost controllable variance

413. In analyzing manufacturing overhead variances, the volume variance is the difference
between the:
A. Amount shown in the flexible budget and the amount shown in the debit side of the
overhead control account
B. Predetermined overhead application rate and the flexible budget application rate times
Chapter 14

actual hours worked


C. Budget allowance based on standard hours allowed for actual production for the period
and the amount budgeted to be applied during the period
D. Actual amount spent for overhead items during the period and the overhead amount
applied to production during the period

414. The variance least significant for purposes of controlling costs is the:
A. Material usage variance
B. Variable overhead efficiency variance
C. Fixed overhead spending variance
D. Fixed overhead volume variance

415. The variance most useful in evaluating plant utilization is the:


A. Variable overhead spending variance
B. Fixed overhead spending variance.
C. Variable overhead efficiency variance
D. Fixed overhead volume variance

416. The choice of production volume as a denominator for calculating its factory overhead
rate
A. Has no effect on the fixed factory overhead rate for applying costs to production
B. Has an effect on the variable factory overhead rate for applying costs to production
C. Has no effect on the fixed factory overhead budget variance
D. Has no effect on the fixed factory overhead production volume variance

417. The budgeted overhead costs for standard hours allowed and the overhead costs
applied to product are the same amount
A. for both variable and fixed overhead costs.
B. only when standard hours allowed is less than normal capacity.
C. for variable overhead costs.
D. for fixed overhead costs.

418. Which department is customarily held responsible for an unfavorable materials usage
variance?
A. Quality control C. Purchasing
B. Engineering D. Production

419. Which of the following should be least considered when deciding whether to investigate
a variance?
A. Whether the variance is favorable or unfavorable
B. Significance of the variance
C. Cost of investigating the variance
D. Trend of the variances over time

420. If the total materials variance (actual cost of materials used compared with the standard
cost of the standard amount of materials required) for a given operation is favorable, why
must this variance be further evaluated as to price and usage?
Chapter 14

A. There is no need to further evaluate the total materials variance if it is favorable


B. Generally accepted accounting principles require that all variances be analyzed in three
stages
C. All variances must appear in the annual report to equity owners for proper disclosure
D. To allow management to evaluate the efficiency of the purchasing and production
functions

421. Which of the following unfavorable cost variances would be directly affected by the
relative position of a production process on a learning curve?
A. Materials mix C. Labor rate
B. Materials price D. Labor efficiency

422. Which of the following is the most probable reason with a company would experience an
unfavorable labor rate variance and a favorable labor efficiency variance?
A. The mix of workers assigned to the particular job was heavily weighted toward the use of
higherly paid, experienced individuals.
B. The mix of workers assigned to the particular job was heavily weighted toward the use of
new, relatively low paid, unskilled workers.
C. Because of the productive schedule, workers from other production areas were assigned
to assist in this particular process.
D. Defective materials caused more labor to be used in order to produce a standard unit.

423. The budget for a given cost during a given period was P1,600,000. The actual cost for
the period was P1,440,000. Considering these facts, it can be said that the plant manager
has done a better than expected job in controlling the cost if:
A. The cost is variable and actual production was 90% of budgeted production
B. The cost is variable and actual production equaled budgeted production
C. The cost is variable and actual production was 80% of budgeted production
D. The cost is discretionary fixed cost and actual production equaled budgeted production

424. The budget variance for fixed factory overhead for the normal-volume, practical-capacity,
and expected-activity levels would be the:
A. Same except for normal volume C. Same except for expected activity
B. Same except for practical capacity D. Same for all three activity levels

425. You have leased a 5,000-gallon storage tank for P5,000 per month. You stored 4,000
gallons of liquid in the tank during the month. The cost of storage was P1.25 per gallon,
rather than P1.00 per gallon based on 5,000 gallon capacity. Therefore, the cost of storing
4,000 gallons was P1,000 more (P.25 x 4,000) in total than if you had stored 5,000 gallons of
liquid in the tank. Which variance is being described?
A. Variable-overhead efficiency variance
B. Fixed-overhead spending variance
C. Variable-overhead spending variance
D. Fixed-overhead volume variance

426. Favorable fixed overhead volume variance occurs if:


A. There is a favorable labor efficiency variance
B. There is a favorable labor rate variance
Chapter 14

C. Production is less than planned


D. Production is greater than planned

427. The unfavorable volume variance may be due to all but which of the following factors?
A. Failure to maintain an even flow of work
B. Machine breakdowns
C. Unexpected increases in the cost of utilities
D. Failure to obtain enough sales orders

428. How will a favorable volume variance affect net income under each of the following
methods?
Absorption Variable
A. Decrease No effect
B. Decrease Increase
C. Increase No effect
D. Increase Decrease

429. Favorable volume variances may be harmful when:


A. Machine repairs cause work stoppages
B. Supervisors fail to maintain an even flow of work
C. Production in excess of normal capacity cannot be sold
D. There are insufficient sales orders to keep the factory operating at normal capacity

430. During 2006, a department’s three-variance overhead standard costing system reported
unfavorable spending and volume variances. The activity level selected for allocating
overhead to the product was based on 80% of practical capacity. It 100% of practical
capacity had been selected instead, how would the reported unfavorable spending and
volume variances be affected?
Spending Volume
Variance Variance
A. Increased Unchanged
B. Increased Increased
C. Unchanged Increased
D. Unchanged Unchanged
Chapter 14

You might also like